LSAT and Law School Admissions Forum

Get expert LSAT preparation and law school admissions advice from PowerScore Test Preparation.

 Administrator
PowerScore Staff
  • PowerScore Staff
  • Posts: 8937
  • Joined: Feb 02, 2011
|
#90591
Complete Question Explanation

Principle, Must Be True. The correct answer choice is (E).

In this stimulus, we get a fairly straightforward principle that we are then asked to find a correct application of. The principle takes the form of a conditional statement: if an action increases the likelihood of physical harm to others AND the action is NOT motivated by a desire to help others then the government is justified in interfering with said action. This can be diagrammed as follows:

ILPH (A) + ~MDHO (A) :arrow: GJI (A)

As is common with stimuli that involve conditional statements, the correct answer might be based off of the contrapositive of the conditional statement, so it would be beneficial to diagram the contrapositive as well. Remember that with conditional statements involving multiple conditions, ANDs become ORs and ORs become ANDs:

~GJI (A) :arrow: ~ILPH (A) or MDHO (A)

Be on the lookout for answer choices that are actually Mistaken Negations or Mistaken Reversals; incorrect answer choices involving these are common with stimuli that involve conditional reasoning.

Answer choice (A): This is a Mistaken Negation. ~ILPH + MDHO :arrow: ~GJI

Answer choice (B): Something that is aesthetically less pleasing does not necessarily increase the likelihood of physical harm to others, so this is not a successful application of the principle. Skip.

Answer choice (C): Similar to (B), we're not told that not wearing a helmet increases the likelihood of physical harm to others, so we can safely skip (C).

Answer choice (D): This answer choice is actually more complicated than the stimulus: it has the structure of premise :arrow: intermediate conclusion :arrow: final conclusion, so it's suspicious right off the bat. And, just like (B) and (C), we're not explicitly told that Zabziew's research increases the likelihood of physical harm to others, just that it hasn't done much to alleviate the harm currently experienced by those suffering from serious illnesses. Skip.

Answer choice (E): This is the correct answer choice. Finally, we get an answer choice that matches every part of our stimulus. We're told that Jill's speech would increase the likelihood of physical harm to others (by causing a riot in which people would have gotten hurt) and that it was done to further her own political ambitions (which we can safely infer is not motivated by a desire to help others). Therefore, we can safely conclude that the government is justified in interfering with her action, specifically by preventing her from giving her speech.
User avatar
 endrm978
  • Posts: 1
  • Joined: Jan 05, 2022
|
#93039
The answer choice is E because the Jill is doing the speech advocating violence specifically to further her career, and her speech is likely to cause physical harm to others by encouraging riots. The reason A is not correct is that Jerry is not increasing harm likeliness and is thinking of others, so the government is quite literally unjustified in interfering, but this is not the principle but rather it's opposite.
 Robert Carroll
PowerScore Staff
  • PowerScore Staff
  • Posts: 1787
  • Joined: Dec 06, 2013
|
#93042
endrm978,

In fact the problem with answer choice (A) is that it involves a Mistaken Negation. In the stimulus, if an action increases the likelihood of physical harm to others and is not motivated by a desire to help others, the government is justified in interfering with it. As a conditional:

(increased risk of harm to others AND motivated by desire to help others) :arrow: govt justified in interfering

In answer choice (A), the sufficient condition is false. This does nothing - trying to infer the negation of the necessary condition would be a Mistaken Negation. So it's not as if answer choice (A) is opposite, it's merely out of scope.

Robert Carroll
User avatar
 nlitovsky
  • Posts: 3
  • Joined: Jun 05, 2022
|
#95690
Hi Robert,

Wouldn't the negation be the following:

govt justified in interfering :arrow: increased risk of harm to others OR motivated by desire to help others
 Rachael Wilkenfeld
PowerScore Staff
  • PowerScore Staff
  • Posts: 1419
  • Joined: Dec 15, 2011
|
#95760
Hi nlitovsky

Absolutely that's the contrapositive! The problem with answer choice (A) is that it's a mistaken reversal of the contrapositive. It says that because Jerry's actions don't cause harm and are made to benefit his children, then the government is unjustified in interfering. Structurally, the conclusion is the necessary condition, which means the government is unjustified is necessary.

Excellent work!
User avatar
 nlitovsky
  • Posts: 3
  • Joined: Jun 05, 2022
|
#95769
Thank you!
User avatar
 blaisebayno
  • Posts: 25
  • Joined: May 24, 2022
|
#96161
My question about E is: why is advocating violence for political means NOT considered helping others? Clearly a person doing such believes it will help some political problems that impacts others negatively.
 Rachael Wilkenfeld
PowerScore Staff
  • PowerScore Staff
  • Posts: 1419
  • Joined: Dec 15, 2011
|
#96183
Hi blaisebayno,

Good question! Here, we have to go based on the information the answer choice gives us. In answer choice (E), we don't have any information to suggest that her underlying desire is to help others. In fact, the answer choice suggests the opposite by stating that she's giving the speech to further her political ambitions. There's nothing in answer choice (E) that we can use to show that her desire to help others motivated her speech. You don't want to read in assumptions about what someone's motivation might be.

Hope that helps!
User avatar
 burneagle9590
  • Posts: 2
  • Joined: Aug 04, 2022
|
#96530
Hi, I'm a bit confused about why A is wrong. Let me see if my understanding of the reasoning it's wrong is correct. A is wrong because it draws it's conclusions from what the contrapositive inspires. However, the negation doesn't necessarily inspire the contrapositive. So if the government is not justified in interfering, then the action doesn't increase the likelihood of harm or it is motivated by a desire to help. BUT the conditions can't inspire the contrapositive, only the contrapositive can inspire the conditions. A) would be correct if the contrapositive was
government not justified :dbl: doesn't increase the likelihood of harm or is motivated by a desire to help.

But A) is incorrect because it simple moves everything over instead of having the conditions be inspired by the negation.

Sorry if my wording is a bit confusing, this is my first time explaining my thought process.
User avatar
 burneagle9590
  • Posts: 2
  • Joined: Aug 04, 2022
|
#96533
Oh wait, I think my reasoning was incorrect. It's wrong because BOTH negations can't be true right? So if his movies were either ONLY focused on helping people or did not cause harm (or intend to). And since both are in the stimulus, the application of the principle was incorrect.

Get the most out of your LSAT Prep Plus subscription.

Analyze and track your performance with our Testing and Analytics Package.